2019年理工学部(数・物・情報・応生・経営工)第1問(1)

理工(数・物・情・応生・経営工)
スポンサーリンク

問題文全文

\(0\leqq \theta \leqq \pi\) のとき 関数

\begin{align}y=\sin^3{\theta}+\cos^3{\theta}-2\sin{2\theta}\end{align}

の最大値と最小値を求めよう.

まず \(x=\sin{\theta}+\cos{\theta}\) とおくと \(\displaystyle x=\sqrt{\fbox{$\hskip0.8emア\hskip0.8em\Rule{0pt}{0.8em}{0.4em}$}}\sin \left(\theta +\frac{\pi}{\fbox{$\hskip0.8emイ\hskip0.8em\Rule{0pt}{0.8em}{0.4em}$}}\right)\) と変形できるので \(x\) のとり得る範囲は \(-\sqrt{\fbox{$\hskip0.8emウ\hskip0.8em\Rule{0pt}{0.8em}{0.4em}$}}\leqq x\leqq \sqrt{\fbox{$\hskip0.8emエ\hskip0.8em\Rule{0pt}{0.8em}{0.4em}$}}\) となる.

次に \(y\) を変形して \(x\) で表すと

\begin{align}y=-\frac{\fbox{$\hskip0.8emオ\hskip0.8em\Rule{0pt}{0.8em}{0.4em}$}}{\fbox{$\hskip0.8emカ\hskip0.8em\Rule{0pt}{0.8em}{0.4em}$}}x^3-\fbox{$\hskip0.8emキ\hskip0.8em\Rule{0pt}{0.8em}{0.4em}$}x^2+\frac{\fbox{$\hskip0.8emク\hskip0.8em\Rule{0pt}{0.8em}{0.4em}$}}{\fbox{$\hskip0.8emケ\hskip0.8em\Rule{0pt}{0.8em}{0.4em}$}}x+2\end{align}

となる. これを微分して因数分解すると

\begin{align}\frac{dy}{dx}=-\frac{3}{2}\left(x-\frac{\fbox{$\hskip0.8emコ\hskip0.8em\Rule{0pt}{0.8em}{0.4em}$}}{\fbox{$\hskip0.8emサ\hskip0.8em\Rule{0pt}{0.8em}{0.4em}$}}\right)(x+\fbox{$\hskip0.8emシ\hskip0.8em\Rule{0pt}{0.8em}{0.4em}$})\end{align}

となる. これらのことより\(,\) \(y\) の最大値は \(\displaystyle \frac{\fbox{$\hskip0.8emスセ\hskip0.8em\Rule{0pt}{0.8em}{0.4em}$}}{\fbox{$\hskip0.8emソタ\hskip0.8em\Rule{0pt}{0.8em}{0.4em}$}},\) 最小値は \(\displaystyle -\fbox{$\hskip0.8emチ\hskip0.8em\Rule{0pt}{0.8em}{0.4em}$}+\frac{1}{2}\sqrt{\fbox{$\hskip0.8emツ\hskip0.8em\Rule{0pt}{0.8em}{0.4em}$}}\) である.

解答

\begin{align}x=\sin{\theta}+\cos{\theta}=\sqrt{2}\sin\left(\theta +\frac{\pi}{4}\right).\end{align}

\(0\leqq \theta \leqq\pi\) より\(,\) \(\displaystyle \frac{\pi}{4}\leqq \theta +\frac{\pi}{4}\leqq \frac{5}{4}\pi\) であるから\(,\)

\begin{align}-\frac{1}{\sqrt{2}}\leqq \sin \left(\theta +\frac{\pi}{4}\right)\leqq 1\end{align}

よって

\begin{align}-1\leqq x\leqq \sqrt{2}.\end{align}

\(x^2=1+2\sin{\theta}\cos{\theta}\) より\(,\)

\begin{align}\sin{\theta}\cos{\theta}=\frac{x^2-1}{2}\end{align}

これを使うと

\begin{align}x^3=\sin^3{\theta}+\cos^3{\theta}+3\sin{\theta}\cos{\theta}(\sin{\theta}+\cos{\theta})\end{align}

\begin{align}=x^3-3\cdot \frac{x^2-1}{2}\cdot x=-\frac{1}{2}x^3+\frac{3}{2}x\end{align}

また\(,\)

\begin{align}2\sin{2\theta}=4\sin{\theta}\cos{\theta}=2(x^2-1)\end{align}

である. 以上より\(,\)

\begin{align}y=-\frac{1}{2}x^3-2x^2+\frac{3}{2}x+2.\end{align}

\begin{align}\frac{dy}{dx}=-\frac{3}{2}x^2-4x+\frac{3}{2}\end{align}

\begin{align}=-\frac{1}{2}(3x^2+8x-3)=-\frac{1}{2}(3x-1)(x+3)\end{align}

\begin{align}=-\frac{3}{2}\left(x-\frac{1}{3}\right)(x+3).\end{align}

よって\(,\) 増減表は以下のようになる.

\begin{align}\begin{array}{c|c|c|c|c|c|}x & -1 & \cdots & \displaystyle \frac{1}{3} & \cdots & \sqrt{2} \\ \hline \displaystyle \frac{dy}{dx} & {} & + & 0 & – & {} \\ \hline y & -1 & \nearrow & \displaystyle \frac{61}{27} & \searrow & \displaystyle -2+\frac{1}{2}\sqrt{2} \\ \end{array}\end{align}

増減表より\(,\)

最大値は \(\displaystyle \frac{61}{27} \).

最小値は \(\displaystyle -2+\frac{1}{2}\sqrt{2}\).

quandle
quandle

最小値については穴の形から即座に \(\displaystyle -2+\frac{1}{2}\sqrt{2}\) と決まりますが\(,\) 実際は -1 との大小を比較しなければなりません.

\begin{align}-1-\left(-2+\frac{1}{2}\sqrt{2}\right)=\frac{2-\sqrt{2}}{2}>0\end{align}

であることから\(,\) 確かに

\begin{align}-1>-2+\frac{1}{2}\sqrt{2}\end{align}

であることが確認できます.

考察:最大値をとるθの値

増減表より\(,\) \(\displaystyle x=\frac{1}{3}\) のときに最大値をとることがわかりました.

与えられている関数は \(\theta\) の関数ですから\(,\) このときの \(\theta\) の値が気になります.

\begin{align}\sqrt{2}\sin (\theta +45^{\circ})=\frac{1}{3}\end{align}

のとき

\begin{align}\sin (\theta +45^{\circ})=\frac{\sqrt{2}}{6}\fallingdotseq 0.2357\end{align}

\(45^{\circ}\leqq \theta +45^{\circ}\leqq 225^{\circ}\) であることに注意すると\(,\) 三角比の表より

\begin{align}\sin 13^{\circ}=0.2250,~\sin 14^{\circ}=0.2419\end{align}

であるから\(,\)

\begin{align}166^{\circ}<\theta +45^{\circ}<167^{\circ}\end{align}

よって\(,\)

\begin{align}121^{\circ}<\theta <122^{\circ}.\end{align}

コメント

タイトルとURLをコピーしました